Howdy, Stranger!

It looks like you're new here. If you want to get involved, click one of these buttons!

PT48, S1, Q10

yhchoi1687yhchoi1687 Alum Member


Hi, I understand why the answer choice (D) is correct, but I have a question on (B).
The stimulus states that since "faculty salaries constitute a small part of the university's expenditure," the administration's explanation is not believable.
So I thought even though faculty salaries constitute a small part of expenditure, if they "significantly" increased after the tuition increase, the faculty's explanation can make sense, so this can be weakening argument.
To counter this weakening argument, I thought (B) can be suggested that increase in faculty salaries is only 5% which is lower than general increase in tuition. This is why I thought (B) can be strengthening argument which can fight back against possible weakening argument.
Probably I thought too complicated in this low number question (supposed to be not this tricky).
But could you explain further why (D) is better answer choice than (B)?

Comments

  • nye8870nye8870 Alum
    1749 karma
    Here's what I see: (B)"With this year’s budget, the university increased the allotment for faculty salaries by 5 percent while tuition was increased by 6 percent." So 5% increase goes to the faculty and 1% to the needy students. That is entirely consistent with the administration's claim. Of course (D) cites all these additional substantial items which will be using/needing more money (cost more).
Sign In or Register to comment.